Diễn Đàn MathScopeDiễn Đàn MathScope
  Diễn Đàn MathScope
Ghi Danh Hỏi/Ðáp Community Lịch

Go Back   Diễn Đàn MathScope > Sơ Cấp > Đại Số và Lượng Giác > Các Bài Toán Đã Được Giải

News & Announcements

Ngoài một số quy định đã được nêu trong phần Quy định của Ghi Danh , mọi người tranh thủ bỏ ra 5 phút để đọc thêm một số Quy định sau để khỏi bị treo nick ở MathScope nhé !

* Nội quy MathScope.Org

* Một số quy định chung !

* Quy định về việc viết bài trong diễn đàn MathScope

* Nếu bạn muốn gia nhập đội ngũ BQT thì vui lòng tham gia tại đây

* Những câu hỏi thường gặp

* Về việc viết bài trong Box Đại học và Sau đại học


Ðề tài đã khoá Gởi Ðề Tài Mới
 
Ðiều Chỉnh Xếp Bài
Old 20-02-2011, 09:26 PM   #916
Nguyenhuyen_AG
+Thành Viên+
 
Nguyenhuyen_AG's Avatar
 
Tham gia ngày: Apr 2010
Bài gởi: 300
Thanks: 35
Thanked 307 Times in 151 Posts
Trích:
Nguyên văn bởi vthiep94 View Post
$\[a^2 + b^2 + c^2 = 3;\] $;a,b,c>0
chứng minh $\[
8\left( {2 - a} \right)\left( {2 - b} \right)\left( {2 - c} \right) \ge (a + bc)(b + ac)(c + ab)
\] $
bài này mình đã lập 1 topic nhưng cách giải sai. Giờ post lại cho mọi người nghiên cứu
Ta có $2(2-a)=4-2a=b^2+c^2+\left (a^2-2a+1 \right )=b^2+c^2+(a-1)^2\ge b^2+c^2 $
Thực hiện tương tự cho hai bất đẳng thức còn lại sau đó nhân theo về với nhau, ta thu được
$8(2-a)(2-b)(2-c)\ge (a^2+b^2)(b^2+c^2)(c^2+c^2) $
Cuối cùng ta chỉ cần chứng minh
$(a^2+b^2)(b^2+c^2)(c^2+c^2)\ge (a+bc)(b+ca)(c+ab) $
Chứng minh bất đẳng thức này không mấy khó khăn.
[RIGHT][I][B]Nguồn: MathScope.ORG[/B][/I][/RIGHT]
 
__________________
Nguyen Van Huyen
Ho Chi Minh City University of Transport
Nguyenhuyen_AG is offline  
The Following User Says Thank You to Nguyenhuyen_AG For This Useful Post:
vthiep94 (20-02-2011)
Old 20-02-2011, 09:33 PM   #917
vthiep94
+Thành Viên+
 
Tham gia ngày: Dec 2009
Bài gởi: 197
Thanks: 185
Thanked 49 Times in 31 Posts
Bạn cm rõ ra đi
[RIGHT][I][B]Nguồn: MathScope.ORG[/B][/I][/RIGHT]
 
vthiep94 is offline  
Old 20-02-2011, 09:40 PM   #918
Nguyenhuyen_AG
+Thành Viên+
 
Nguyenhuyen_AG's Avatar
 
Tham gia ngày: Apr 2010
Bài gởi: 300
Thanks: 35
Thanked 307 Times in 151 Posts
Trích:
Nguyên văn bởi vthiep94 View Post
Bạn cm rõ ra đi
Bạn hãy thử chứng minh nó mình đã gợi ý là nó không khó rồi mà, nếu có thắc mắc gì thì hãy hỏi, mọi người sẽ nhiệt tình trả lời cho bạn.
------------------------------
Ta phân tích rõ hơn cho bài toán.
Vì $\frac{a^2+b^2}{a+b}=a+b-\frac{2ab}{a+b} $ nên ta có thể viết bất đẳng thức lại như sau
$2(a+b+c)\left ( a+b+c-\frac{ab}{a+b}-\frac{bc}{b+c}-\frac{ca}{c+a} \right )\le 3(a^2+b^2+c^2) $
Tương đương với
$3(a^2+b^2+c^2)+2(a+b+c)\left ( \frac{ab}{a+b}+\frac{bc}{b+c}+\frac{ca}{c+a} \right )\ge 2(a+b+c)^2 $
$a^2+b^2+c^2+\sum 2ab\left ( 1+\frac{c}{a+b} \right )\ge 4(ab+bc+ca) $
$a^2+b^2+c^2+ 2abc\left (\frac{1}{a+b}+\frac{1}{b+c}+\frac{1}{c+a} \right) \ge 4(ab+bc+ca) $

[RIGHT][I][B]Nguồn: MathScope.ORG[/B][/I][/RIGHT]
 
__________________
Nguyen Van Huyen
Ho Chi Minh City University of Transport

thay đổi nội dung bởi: Nguyenhuyen_AG, 20-02-2011 lúc 10:04 PM Lý do: Tự động gộp bài
Nguyenhuyen_AG is offline  
Old 20-02-2011, 10:22 PM   #919
MathForLife
+Thành Viên+
 
Tham gia ngày: Sep 2010
Đến từ: CT force
Bài gởi: 731
Thanks: 603
Thanked 425 Times in 212 Posts
Trích:
Nguyên văn bởi Quydo View Post
Cho a,b,c>0 CMR:
$a^2b^2(a-b)^2+b^2c^2(b-c)^2+c^2a^2(c-a)^2 \geq [(a-b)(b-c)(c-a)]^2 $
Sử dụng hằng đẳng thức:

$(a-b)(b-c)(c-a)=ab(a-b)+bc(b-c)+ca(c-a) $
Ta chỉ cần chứng minh:

$b(a-b)(b-c)+c(b-c)(c-a)+a(c-a)(a-b)\le 0 $

Đây chính là bất đẳng thức Schur bậc 3.
------------------------------
Trích:
Nguyên văn bởi conan236 View Post
Hai bất đẳng thức 6 biến
2. Cho $a,b,c,d,x,y,z $ là các số thực thỏa : $(a+b+c)(x+y+z)=3 $ và $(a^2+b^2+c^2)(x^2+y^2+z^2)=4 $. Chứng minh rằng :
$ax+by+cz \geq 0 $
Đặt $t=\sqrt[4]{\frac{a^2+b^2+c^2}{x^2+y^2+z^2}} $ thì:
$a^2+b^2+c^2=2t^2 $
$x^2+y^2+z^2=\frac{2}{t^2} $
Bất đẳng thức cần chứng minh tương đương:
$\left ( \frac{a}{t}+tx \right )^2+\left ( \frac{b}{t}+ty \right )^2+\left ( \frac{c}{t}+tz \right )^2\ge 4 $ (*)
Ta có:
$VT(*)\ge \frac{1}{3}\left ( \frac{a+b+c}{t}+t(x+y+z) \right )^2\ge \frac{4}{3}(a+b+c)(x+y+z)=4 $
Vậy bất đẳng thức dc chứng minh.
[RIGHT][I][B]Nguồn: MathScope.ORG[/B][/I][/RIGHT]
 

thay đổi nội dung bởi: MathForLife, 20-02-2011 lúc 10:33 PM Lý do: Tự động gộp bài
MathForLife is offline  
The Following 2 Users Say Thank You to MathForLife For This Useful Post:
daylight (20-02-2011), Quydo (20-02-2011)
Old 21-02-2011, 01:07 AM   #920
conan236
+Thành Viên+
 
Tham gia ngày: Dec 2007
Đến từ: Đà Nẵng
Bài gởi: 287
Thanks: 17
Thanked 104 Times in 43 Posts
Trích:
Nguyên văn bởi manhnguyen94 View Post
Cho a,b,c là 3 cạnh của 1 tam giác, chứng minh rằng:
$(2 \sum a^2(b+c)- 2 \sum a^3 +3abc)abc \geq (a+b+c)(a+b-c)(b+c-a)(c+a-b)(ab+ba+ca) $
Áp dụng đẳng thức :
$(b+c-a)(c+a-b)(a+b-c) = \sum a^2(b+c) - \sum a^3 -2abc $
Ta suy ra bđt cần chứng minh tương đương với :
$2abc\prod (b+c-a) + 7a^2b^2c^2 \geq (a+b+c)(a+b-c)(b+c-a)(c+a-b)(ab+ba+ca) $
Tuy nhiên ta có một số đẳng thức trong tam giác sau đây :
$+16S^2 = (a+b+c)(a+b-c)(b+c-a)(c+a-b) $
$+abc=4SR, S=pr $
Trong đó $S,p,R,r $ lần lượt là kí hiệu của diện tích, nửa chu vi, bán kính đường tròn ngoại tiếp và nội tiếp tam giác.
Từ các đẳng thức trên ta dễ dàng đưa bất đẳng thức về dưới dạng :
$4\frac{r}{R} + 7 \geq 4\sum sinAsinB $
$\Leftrightarrow 4\sum cos A +3 \geq 4\sum sinAsinB $
$\Leftrightarrow 4\sum cosAcosB \leq 3 $
Mà bất đẳng thức là khá đơn giản bởi vì ta có :
$4\sum cosAcosB \leq 4\frac{(cosA+cosB+cosC)^2}{3} \leq \frac{4}{3}.\left (\frac{3}{2} \right)^2 = 3 $
Vậy ta có đpcm.
[RIGHT][I][B]Nguồn: MathScope.ORG[/B][/I][/RIGHT]
 
__________________
TOÁN HỌC LÀ CUỘC SỐNG CỦA TÔI
conan236 is offline  
The Following 3 Users Say Thank You to conan236 For This Useful Post:
daylight (21-02-2011), long_chau2010 (21-02-2011), manhnguyen94 (21-02-2011)
Old 21-02-2011, 01:17 AM   #921
daylight
+Thành Viên+
 
daylight's Avatar
 
Tham gia ngày: Dec 2009
Đến từ: Ha Noi
Bài gởi: 551
Thanks: 877
Thanked 325 Times in 188 Posts
Cho $a,b,c \ge 0 $. Chứng minh rằng :
$\sqrt[3]{a+\sqrt[3]{a}}+\sqrt[3]{b+\sqrt[3]{b}}+\sqrt[3]{c+\sqrt[3]{c}} \le \sqrt[3]{a+\sqrt[3]{b}}+\sqrt[3]{b+\sqrt[3]{c}}}+\sqrt[3]{c+\sqrt[3]{a}} $


[RIGHT][I][B]Nguồn: MathScope.ORG[/B][/I][/RIGHT]
 

thay đổi nội dung bởi: daylight, 21-02-2011 lúc 01:31 AM
daylight is offline  
The Following User Says Thank You to daylight For This Useful Post:
khaitang1234 (21-02-2011)
Old 21-02-2011, 12:05 PM   #922
birain9x
+Thành Viên+
 
birain9x's Avatar
 
Tham gia ngày: Feb 2011
Bài gởi: 119
Thanks: 28
Thanked 41 Times in 23 Posts
Cho a,b,c>0 tm $\frac{1}{a^2+47}+\frac{1}{b^2+47}+\frac{1}{c^2+47} =\frac{1}{24} $.CMR $a+b+c\geq 15 $
[RIGHT][I][B]Nguồn: MathScope.ORG[/B][/I][/RIGHT]
 
birain9x is offline  
The Following User Says Thank You to birain9x For This Useful Post:
khaitang1234 (21-02-2011)
Old 21-02-2011, 05:26 PM   #923
khaitang1234
+Thành Viên+
 
Tham gia ngày: Nov 2010
Bài gởi: 81
Thanks: 86
Thanked 96 Times in 53 Posts
Gửi tin nhắn qua Yahoo chát tới khaitang1234
Cho $a,b,c>0 $ thoả mãn $a+b+c=1 $. Chứng minh:
$\frac{1}{a^{2}(1+a)}+\frac{1}{b^{2}(1+b)}+\frac{1} {c^{2}(1+c)}\geq \frac{3}{4abc} $
[RIGHT][I][B]Nguồn: MathScope.ORG[/B][/I][/RIGHT]
 
khaitang1234 is offline  
Old 21-02-2011, 05:49 PM   #924
conan236
+Thành Viên+
 
Tham gia ngày: Dec 2007
Đến từ: Đà Nẵng
Bài gởi: 287
Thanks: 17
Thanked 104 Times in 43 Posts
Trích:
Nguyên văn bởi khaitang1234 View Post
Cho $a,b,c>0 $ thoả mãn $a+b+c=1 $. Chứng minh:
$\frac{1}{a^{2}(1+a)}+\frac{1}{b^{2}(1+b)}+\frac{1} {c^{2}(1+c)}\geq \frac{3}{4abc} $
Áp dụng bđt BCS ta có :
$\sum \frac{bc}{a(1+a)}=\sum\frac{b^2c^2}{abc(1+a)} \geq \frac{(\sum bc)^2}{4abc}\geq \frac{3abc(a+b+c)}{4abc} =\frac{3}{4} $
Vậy nên :
$\frac{1}{a^{2}(1+a)}+\frac{1}{b^{2}(1+b)}+\frac{1} {c^{2}(1+c)}\geq \frac{3}{4abc} $
------------------------------
Trích:
Nguyên văn bởi conan236 View Post
Một bài bđt tam giác khác :
Cho $a,b,c $ là ba cạnh tam giác. Chứng minh rằng :
$\sum \frac{bc\sqrt{3a^2+2b^2+2c^2+2bc}}{b+c-a} \geq 3(ab+bc+ca) $

-----------------------------
1. Cho $a,b,c>0 $. Chứng minh rằng :
$2(a^2+1)(b^2+1)(c^2+1) \geq abc(a+1)(b+1)(c+1) $
2. Cho tam giác $ABC $ nhọn. Chứng minh rằng :
$\frac{cosAcosB}{sin2C}+\frac{cosBcosC}{sin2A}+ \frac{cosCcosA}{sin2B} \geq \frac{\sqrt{3}}{2} $
3. Cho $x_1,x_2,...,x_n $ là các số thực. Chứng minh rằng :
$\sum_{1\leq i \leq j\leq n} |x_i+x_j| \geq \frac{n-2}{2} \sum_{i=1}^{n} |x_i| $

[RIGHT][I][B]Nguồn: MathScope.ORG[/B][/I][/RIGHT]
 
__________________
TOÁN HỌC LÀ CUỘC SỐNG CỦA TÔI

thay đổi nội dung bởi: conan236, 21-02-2011 lúc 06:08 PM Lý do: Tự động gộp bài
conan236 is offline  
The Following User Says Thank You to conan236 For This Useful Post:
dandoh221 (21-02-2011)
Old 21-02-2011, 05:56 PM   #925
yeu
+Thành Viên+
 
Tham gia ngày: Sep 2009
Bài gởi: 54
Thanks: 6
Thanked 8 Times in 7 Posts
Bất đẳng thức

1)Cho $x_i \geqslant 1;i = 1,2,....,n $. CMR
$\frac{1}{{1 + x_1 }} + \frac{1}{{1 + x_2 }} + ..... + \frac{1}{{1 + x_n }}\geqslant \frac{n}{{1 + \sqrt[n]{{x_1 x_2 .....x_n }}}} $

2)Cho $a,b,c \geqslant 0 ,a + b + c = 3 $ .CMR
$\left( {a^3 + b^3 + c^3 } \right)\left( {a^3 b^3 + b^3 c^3 + c^3 a^3 } \right) \leqslant 36\left( {ab + bc + ac} \right) $

4)Cho $a,b,c > 0 $ , $abc = 1 $ .CMR
$\frac{1}{a} + \frac{1}{b} + \frac{1}{c} + \frac{6}{{a + b + c}} \geqslant 5 $

3)Cho $a,b,c\ > 0 $ .CMR
$2\left ( a^{2}+b^{2}+c^{2} \right )+3\sqrt[3] {a^2.b^2.c^2 }\geq \left ( a+b+c\right )^2 $

Rất mong các bạn giải nó,càng nhiều cách càng tôt
[RIGHT][I][B]Nguồn: MathScope.ORG[/B][/I][/RIGHT]
 
yeu is offline  
Old 21-02-2011, 07:09 PM   #926
IMO 2010
+Thành Viên+
 
IMO 2010's Avatar
 
Tham gia ngày: Nov 2010
Bài gởi: 22
Thanks: 1,516
Thanked 4 Times in 4 Posts
1 bất đẳng thức

Cho $a > 0 $. CMR: $a + \left | 2a^{2} -1\right | \geq \frac{1}{2} $
------------------------------
[RIGHT][I][B]Nguồn: MathScope.ORG[/B][/I][/RIGHT]
 
__________________
IMO
.................................................. .................................................. .................................................. .................................................. .................................................. .................................................. .................................................. .................................................. .................................................. ......

thay đổi nội dung bởi: IMO 2010, 21-02-2011 lúc 11:43 PM Lý do: Tự động gộp bài
IMO 2010 is offline  
Old 21-02-2011, 08:10 PM   #927
dandoh221
+Thành Viên+
 
dandoh221's Avatar
 
Tham gia ngày: Nov 2009
Bài gởi: 47
Thanks: 19
Thanked 18 Times in 14 Posts
Trích:
Nguyên văn bởi yeu View Post
3)Cho $a,b,c\ > 0 $ .CMR
$2\left ( a^{2}+b^{2}+c^{2} \right )+3\sqrt[3] {a^2.b^2.c^2 }\geq \left ( a+b+c\right )^2 $

Rất mong các bạn giải nó,càng nhiều cách càng tôt
giải đc cách dở lắm :
cho abc = 1. Ta cần chứng minh$ a^2+b^2+c^2 + 3 \ge 2(ab+bc+ac) $
$f(a,b,c) = a^2+b^2-2ab-c(a+b) +c^2-3 $
giả sử c = min{a,b,c}
đặt \sqrt{ab} = t
$f(a,b,c) - f(t,t,c) = (\sqrt{a} -\sqrt{b})^2(a+b+2\sqrt{ab} - 2c) \ge 0 $
thay $t = \sqrt{\frac{1}{c}} $
$f(t,t,c) = (\sqrt{c} - 1})({c\sqrt{c} - c + \sqrt{c} - 5) \ge 0 $

[RIGHT][I][B]Nguồn: MathScope.ORG[/B][/I][/RIGHT]
 
__________________
graciás por favor me, graciás amigos !

thay đổi nội dung bởi: dandoh221, 21-02-2011 lúc 08:12 PM
dandoh221 is offline  
Old 21-02-2011, 08:31 PM   #928
magic.
+Thành Viên+
 
Tham gia ngày: Aug 2010
Bài gởi: 213
Thanks: 107
Thanked 140 Times in 84 Posts
Trích:
Nguyên văn bởi yeu View Post



3)Cho $a,b,c\ > 0 $ .CMR
$2\left ( a^{2}+b^{2}+c^{2} \right )+3\sqrt[3] {a^2.b^2.c^2 }\geq \left ( a+b+c\right )^2 $
Dùng Schur thôi

BĐT $\Leftrightarrow a^{2}+b^{2}+c^{2} +3\sqrt[3] {a^2.b^2.c^2 }\geq 2(ab+ac+bc) $

Vì $3\sqrt[3] {a^2.b^2.c^2 } \ge \frac{9abc}{a+b+c} $

Mà $\Leftrightarrow a^{2}+b^{2}+c^{2} + \frac{9abc}{a+b+c} \geq 2(ab+ac+bc) $ (Schur)

Nên ta có đpcm.
------------------------------
Trích:
Nguyên văn bởi yeu View Post

4)Cho $a,b,c > 0 $ , $abc = 1 $ .CMR
$\frac{1}{a} + \frac{1}{b} + \frac{1}{c} + \frac{6}{{a + b + c}} \geqslant 5 $


Bài này dùng $pqr $ và đánh giá Schur.Bạn Daylight đã tưng giải ở đâu đó.Bạn có thể tìm ở mục bài bạn ấy đã post
------------------------------
Trích:
Nguyên văn bởi conan236 View Post
-
1. Cho $a,b,c>0 $. Chứng minh rằng :
$2(a^2+1)(b^2+1)(c^2+1) \geq abc(a+1)(b+1)(c+1) $
Kết quả mạnh hơn Vasc:

Cho $a,b,c>0 $. Chứng minh rằng :
$2(a^2+1)(b^2+1)(c^2+1) \geq (abc+1)(a+1)(b+1)(c+1) $


------------------------------
Trích:
Nguyên văn bởi yeu View Post
1)Cho $x_i \geqslant 1;i = 1,2,....,n $. CMR
$\frac{1}{{1 + x_1 }} + \frac{1}{{1 + x_2 }} + ..... + \frac{1}{{1 + x_n }}\geqslant \frac{n}{{1 + \sqrt[n]{{x_1 x_2 .....x_n }}}} $

[RIGHT][I][B]Nguồn: MathScope.ORG[/B][/I][/RIGHT]
 
__________________
Peace195

thay đổi nội dung bởi: magic., 21-02-2011 lúc 08:56 PM Lý do: Tự động gộp bài
magic. is offline  
Old 22-02-2011, 05:15 PM   #929
Quydo
+Thành Viên+
 
Quydo's Avatar
 
Tham gia ngày: Oct 2010
Bài gởi: 182
Thanks: 143
Thanked 79 Times in 55 Posts
Giải bằng pp tiếp tuyến

Cho $a^2+b^2+c^2=1 $
Tìm max $P=\sum \frac{1}{1-ab} $
P/s:Giải hộ mình với sao biến đổi mãi ko ra
[RIGHT][I][B]Nguồn: MathScope.ORG[/B][/I][/RIGHT]
 
__________________
MH
MH
Quydo is offline  
Old 23-02-2011, 08:58 AM   #930
mathematician
+Thành Viên+
 
mathematician's Avatar
 
Tham gia ngày: Jan 2011
Đến từ: Hall of Fame
Bài gởi: 65
Thanks: 29
Thanked 30 Times in 10 Posts
Trích:
Nguyên văn bởi Quydo View Post
Cho $a^2+b^2+c^2=1 $
Tìm max $P=\sum \frac{1}{1-ab} $
P/s:Giải hộ mình với sao biến đổi mãi ko ra
Chính là bài này sau khi đã đổi biến:
[Only registered and activated users can see links. ]
[RIGHT][I][B]Nguồn: MathScope.ORG[/B][/I][/RIGHT]
 
__________________
"By denying scientific principles, one may maintain any paradox" - Galileo Galilei
mathematician is offline  
The Following User Says Thank You to mathematician For This Useful Post:
Quydo (23-02-2011)
Ðề tài đã khoá Gởi Ðề Tài Mới

Bookmarks

Tags
bất đẳng thức


Quuyền Hạn Của Bạn
You may not post new threads
You may not post replies
You may not post attachments
You may not edit your posts

BB code is Mở
Smilies đang Mở
[IMG] đang Mở
HTML đang Tắt

Chuyển đến


Múi giờ GMT. Hiện tại là 11:13 AM.


Powered by: vBulletin Copyright ©2000-2024, Jelsoft Enterprises Ltd.
Inactive Reminders By mathscope.org
[page compression: 104.99 k/122.29 k (14.15%)]